08-nguyễn anh huân

cho x>0 và y>0 cm (x+y)(1/x+1/y) lớn hơn hoặc bằng 4

 

Nguyễn Ngọc Huy Toàn
17 tháng 5 2022 lúc 20:39

Áp dụng BĐT AM-GM,ta có:

\(\dfrac{1}{x}+\dfrac{1}{y}\ge\dfrac{4}{x+y}\)

\(\Rightarrow\left(x+y\right)\left(\dfrac{1}{x}+\dfrac{1}{y}\right)\ge\dfrac{4\left(x+y\right)}{x+y}\)

\(\Leftrightarrow\left(x+y\right)\left(\dfrac{1}{x}+\dfrac{1}{y}\right)\ge4\) ( đfcm )

 

 

 

Bình luận (2)
hacker nỏ
17 tháng 5 2022 lúc 20:50

Có: \(\left(x+y\right)\left(\dfrac{1}{x}+\dfrac{1}{y}\right)\ge4\)\(\dfrac{1}{x}+\dfrac{1}{y}\ge\dfrac{4}{x+y}\)\(\dfrac{y+x}{xy}\ge\dfrac{4}{x+y}\)

\(\dfrac{\left(x+y\right)\left(x+y\right)}{xy\left(x+y\right)}\ge\dfrac{4xy}{xy\left(x+y\right)}\)\(\left(x+y\right)^2\ge4xy\)\(x^2+2xy+y^2\ge4xy\)

\(x^2-4xy+2xy+y^2\ge0\)\(x^2-2xy+y^2\ge0\)\(\left(x-y\right)^2\ge0\) luôn đúng 

Bình luận (0)

Các câu hỏi tương tự
Ánh Trần
Xem chi tiết
Quang Boy
Xem chi tiết
Nguyễn Thị Dung
Xem chi tiết
Diem Quynh
Xem chi tiết
Lê Hồng Ngọc
Xem chi tiết
Nguyễn Hoàng
Xem chi tiết
Nguyễn Tiến Hiệp
Xem chi tiết
Nguyễn Hoàng Phương Nhàn
Xem chi tiết
Nguyễn Khánh Linh
Xem chi tiết